Finding Solutions to DE using Laplace Transform

Click For Summary
SUMMARY

The discussion focuses on solving the differential equation y'' - 6y' + 9y = f(t) using the Laplace Transform. The function f(t) is defined piecewise with Heaviside step functions, leading to the equation y'' - 6y' + 9y = u1 - u3. The solution provided is y(t) = 2e^t + u1(1/9 - 1/9e^(3(t-1)) + 1/3(t-1)e^(3(t-1))). The user questions the absence of u3 in the final answer and seeks clarification on the initial conditions y(0) = 1 and y'(0) = 2, which are crucial for solving the differential equation accurately.

PREREQUISITES
  • Understanding of differential equations, specifically second-order linear equations.
  • Familiarity with the Laplace Transform and its properties.
  • Knowledge of Heaviside step functions and their application in piecewise functions.
  • Ability to apply initial conditions to solve differential equations.
NEXT STEPS
  • Study the application of the Laplace Transform on piecewise functions.
  • Learn about the properties and applications of Heaviside step functions in differential equations.
  • Explore methods for solving second-order linear differential equations with constant coefficients.
  • Review the significance of initial conditions in determining unique solutions for differential equations.
USEFUL FOR

Students and educators in mathematics, particularly those focusing on differential equations, as well as engineers and physicists applying these concepts in practical scenarios.

manenbu
Messages
101
Reaction score
0

Homework Statement



y'' -6y' + 9y = f(t)

f(t)=
0, 0<t<1
1, 1<t<3
0, t>3

Homework Equations





The Attempt at a Solution



Turning it to heaviside functions I get:
y'' -6y' + 9y = u1 - u3

and I solve.

in the answers it should be:
y(t) = 2tet + u1(1/9 - 1/9e3(t-1)+ 1/3(t-1)e3(t-1)).

where did u3 go in the answers? why doesn't it appear? the u1 I can get just like that, but I can't seem to figure out where the u3 went and why there is a 2tet in there.
 
Physics news on Phys.org
It looks like we're missing some information. Were there additional conditions given? (e.g. y(0) = ? and y'(0) = ?)

Also, what did the DE look like after you applied the Laplace transform?
 
it doesn't look as though you changed anything by putting it in unit step form.
 
Sorry, forgot.
y(0) = 1, y'(0) = 2.
 
Question: A clock's minute hand has length 4 and its hour hand has length 3. What is the distance between the tips at the moment when it is increasing most rapidly?(Putnam Exam Question) Answer: Making assumption that both the hands moves at constant angular velocities, the answer is ## \sqrt{7} .## But don't you think this assumption is somewhat doubtful and wrong?

Similar threads

  • · Replies 7 ·
Replies
7
Views
2K
  • · Replies 10 ·
Replies
10
Views
2K
  • · Replies 4 ·
Replies
4
Views
3K
  • · Replies 2 ·
Replies
2
Views
2K
Replies
2
Views
2K
  • · Replies 1 ·
Replies
1
Views
967
Replies
3
Views
2K
  • · Replies 1 ·
Replies
1
Views
1K
  • · Replies 3 ·
Replies
3
Views
3K
  • · Replies 3 ·
Replies
3
Views
1K